```{r data generation, echo = FALSE, results = "hide"} n <- sample(8:15, 1) y <- rnorm(n, runif(1, 4940, 4990), runif(1, 30, 50)) alpha <- sample(c(0.1, 0.05, 0.01), 1) Mean <- round(mean(y), digits = 1) Var <- round(var(y), digits = 1) sd <- sqrt(Var/n) fact <- round(qt(1 - alpha/2, df = n - 1), digits = 4) facn <- round(qnorm(1 - alpha/2), digits = 4) LBt <- round(Mean - fact * sd, digits = 3) UBt <- round(Mean + fact * sd, digits = 3) LBn <- round(Mean - facn * sd, digits = 3) UBn <- round(Mean + facn * sd, digits = 3) ## use extended Moodle processing to award 100% for correct solution based on ## t quantiles and 50% for the solution based on normal quantiles ## ## this can be handled as a "verbatim" solution, directly including Moodles ## cloze type: ## ":NUMERICAL:=2.228:0.01~%50%1.960:0.01#Normal-based instead of t-based interval." ## where 2.228 is the correct and 1.960 the partially correct solution, ## the tolerance is 0.01 in both cases, and a comment is supplied at the end. ## More details: https://docs.moodle.org/35/en/Embedded_Answers_(Cloze)_question_type ## solution template (note: % have to be escaped as %% for sprintf) sol <- ":NUMERICAL:=%s:0.1~%%50%%%s:0.1#Normal-based instead of t-based interval; for small samples, intervals based on the normal approximation are too narrow." ## insert correct and partially correct solutions sol <- sprintf(sol, c(LBt, UBt), c(LBn, UBn)) ``` Question ======== It is suspected that a supplier systematically underfills 5 l canisters of detergent. The filled volumes are assumed to be normally distributed. A small sample of $`r n`$ canisters is measured exactly. This shows that the canisters contain on average $`r Mean`$ ml. The sample variance $s^2_{n-1}$ is equal to $`r Var`$. Determine a $`r 100 * (1 - alpha)`\%$ confidence interval for the average content of a canister (in ml). Answerlist ---------- * What is the lower confidence bound? * What is the upper confidence bound? Solution ======== The $`r 100 * (1 - alpha)`\%$ confidence interval for the average content $\mu$ in ml is given by: $$ \begin{aligned} & \left[\bar{y} \, - \, t_{n-1;`r 1-alpha/2`}\sqrt{\frac{s_{n-1}^2}{n}}, \; \bar{y} \, + \, t_{n-1;`r 1-alpha/2`}\sqrt{\frac{s_{n-1}^2}{n}}\right] \\ &= \left[ `r Mean` \, - \, `r fact`\sqrt{\frac{`r Var`}{`r n`}}, \; `r Mean` \, + \, `r fact`\sqrt{\frac{`r Var`}{`r n`}}\right] \\ &= \left[`r LBt`, \, `r UBt`\right]. \end{aligned} $$ Answerlist ---------- * The lower confidence bound is $`r LBt`$. * The upper confidence bound is $`r UBt`$. Meta-information ============ extype: cloze exclozetype: verbatim|verbatim exsolution: `r sol[1]`|`r sol[2]` exname: Confidence interval extol: 0.01